pharma q bank 80 p partv 2 23 Flashcards

1
Q

(Question ID: 123) A 50-year-old man comes to the physician’s office expressing concern for his “ticker.” His father just died of a heart attack. The physicians had told his father that he had too much bad cholesterol. The patient wants to know how to reduce his risk of having a heart attack. His blood pressure is 150/95 mm Hg, and he is obese. He reports that he was taking some kind of statin before, but it made his legs cramp. The physician offers him an alternative drug to lower his bad cholesterol. This drug works by which of the following mechanisms?
a) Binding and excretion of bile-soluble acids
b) Decreased peripheral lipolysis
c) Inhibition of the rate-limiting enzyme of cholesterol formation
d) Retention of bile acid resins in hepatocytes
e) Suppressing liver gluconeogenesis

A

[Y]Binding and excretion of bile-soluble acids

How well did you know this?
1
Not at all
2
3
4
5
Perfectly
2
Q

(Question ID: 124) A 33-year-old woman presents to the emergency department with a 45-minute episode of sudden onset of palpitations. ECG demonstrates a regular narrow complex tachycardia. Upon discharge she is started on arrhythmic therapy with β blockers to suppress recurrences of supraventricular tachycardia. What β blocker used to treat arrhythmia prolongs the repolarization phase of the cardiac action potential?
a) Acebutolol
b) Esmolol
c) Metoprolol
d) Propranolol
e) Sotalol

A

[Y]Sotalol

How well did you know this?
1
Not at all
2
3
4
5
Perfectly
3
Q

(Question ID: 125) A 60-year-old man with Wolff-Parkinson-White syndrome currently receiving pharmacologic treatment presents to the clinic with fatigue and a 2.3-kg (5-lb) weight gain over the past 3 months. Physical examination results are normal. Laboratory studies show:
WBC count: 7000/mm3
Hematocrit: 44%
Hemoglobin: 15.6 g/dL
Platelet count: 270,000/mm3
Na+: 136 mEq/L
K+: 3.8 mEq/L
Cl ־:100 mEq/L
HCo3 ־: 24 mEq/L
Blood urea nitrogen: 12 mg/dL
Creatinine: 1.1 mg/dL
Thyroid-stimul3ting hormone: 23 U/mL
Thyroxine: 2.0 pg/dL
Prothrombin time: 12 seconds
Activated partial thromboplastin time: 30 seconds
The patient did not bring a list of his current medications.
a) Amiodarone
b) Enoxaparin
c) Metoprolol
d) Quinidine
e) Warfarin

A

[Y]Amiodarone

How well did you know this?
1
Not at all
2
3
4
5
Perfectly
4
Q

(Question ID: 126) A 35-year-old pregnant woman has severe, drug-resistant hypertension, aS well aS a history of cardiac dysrhythmias. Despite her cardiovascular conditions, the patient still has normal kidney function. An internist decides to try a potent vasodilator to control the patient’s hypertension. He also decides to co¬administer a β-blocker to prevent one of the more worrisome possible adverse effects of the medication. Which vasodilator was administered to the patient?
a) Amiodarone
b) Captopril
c) Hydralazine
d) Losartan
e) Methyldopa

A

[Y]Hydralazine

How well did you know this?
1
Not at all
2
3
4
5
Perfectly
5
Q

(Question ID: 127) An 82-year-old man is found unconscious in the park and is brought by ambulance to the nearest emergency department. Chart review reveals coronary artery disease, congestive heart failure, and moderate dementia; he has been taking digitalis for nearly 20 years. An ECG demonstrates atrial fibrillation Drug X is given, which successfully stops the atrial fibrillation, but 30 minutes later the patient exhibits nausea, vomiting, and atrial tachycardia with atrioventricular block. What is drug X?
a) Antidigitalis Fab
b) Lidocaine
c) Phenytoin
d) Quinidine
e) Tocainide

A

[Y]Quinidine

How well did you know this?
1
Not at all
2
3
4
5
Perfectly
6
Q

(Question ID: 128) A 45-year-old man with essential hypertension presents to the emergency department because of progressive muscle weakness. His blood pressure is 140/90 mm Hg, pulse is 85/min, respiratory rate is 13/min, and temperature is 37.C (98.6.F). ECG reveals peaked T waves and prolonged PR intervals. Which of the following medications could be the underlying cause of the changes noted on this patient’s ECG?
a) Acetazolamide
b) Furosemide
c) Hydrochlorothiazide
d) Mannitol
e) Spironolactone

A

[Y]Spironolactone

How well did you know this?
1
Not at all
2
3
4
5
Perfectly
7
Q

(Question ID: 129) A 52-year-old man is brought, unresponsive, to the emergency department by emergency medical services. A stat ECG shows ventricular fibrillation. Electrical defibrillation is initiated but is unsuccessful. The use of antiarrhythmic drugs to treat this shock-refractory ventricular fibrillation is discussed, but there are concerns about side effects of pulmonary fibrosis in this patient, who has a history of lung disease. What is the mechanism of action of the antiarrhythmic drug that was most likely administered?
a) Accelerates depolarization in high-frequency depolarizing cells
b) Prolongs depolarization via sodium channel blockade
c) Prolongs the repolarization phase via potassium channel blockade
d) Reduces calcium influx in depolarizing cells
e) Reduces sympathetic stimulation of myocardial cells

A

[Y]Prolongs the repolarization phase via potassium channel blockade

How well did you know this?
1
Not at all
2
3
4
5
Perfectly
8
Q

(Question ID: 130) A 45-year-old man presents to the clinic for an annual physical. He has a history of hypertension, and his medications include hydrochlorothiazide and metoprolol. At the clinic his blood pressure reading is 168/95 mm Hg. On a follow-up visit 3 weeks later his blood pressure remains elevated at 160/90 mm Hg. He is subsequently prescribed a transdermal medication to provide additional control of his hypertension. Which of the following describes the mechanism of action of this transdermal medication?
a) α1-Adrenergic receptor antagonist
b) α2-Adrenergic receptor agonist
c) β1-Adrenergic receptor antagonist
d) α2-Adrenergic receptor antagonist
e) Inhibitor of angiotensin-converting enzyme

A

[Y]α2-Adrenergic receptor agonist

How well did you know this?
1
Not at all
2
3
4
5
Perfectly
9
Q

(Question ID: 131) A patient presents to the emergency department with a severe headache, palpitations, and elevated blood pressure. He is found to have elevated urinary vanillylmandelic acid levels. He is diagnosed with a pheochromocytoma with predominantly elevated norepinephrine levels. Which of the following agents will antagonize both the vascular and cardiac actions of norepinephrine?
a) Atenolol
b) Doxazosin
c) Esmolol
d) Isoproterenol
e) Labetalol

A

[Y]Labetalol

How well did you know this?
1
Not at all
2
3
4
5
Perfectly
10
Q

(Question ID: 132) A 64-year-old man has a history of shortness of breath and edema in the lower extremities. One of the medications he takes is a sodium-potassium- chloride co־transport inhibitor. Which of the following is an indication for taking this type of medication?
a) Dilated cardiomyopathy
b) Hypertrophic obstructive cardiomyopathy
c) Hypokalemia
d) Hypotension
e) Primary hypoparathyroidism

A

[Y]Dilated cardiomyopathy

How well did you know this?
1
Not at all
2
3
4
5
Perfectly
11
Q

(Question ID: 133) A 54-year-old woman presents to the emergency department because of severe рain between her shoulder blades of 6 minutes’ duration. She states that she vomited on the way to the hospital. Her hands are clammy and she appears very anxious. An ECG from triage is shown in the image. The patient is given a medication that rapidly relieves her pain. What is the mechanism of action of this medication?
a) Inhibition of active chloride reabsorption at the early distal tubule via the sodium-chloride co־transporter
b) Inhibition of angiotensin-converting enzyme
c) Inhibition of the sodium/potassium adenosinetriphosphatase pump and increase in the atrioventricular node refractory period
d) Inhibition of transmembrane influx of calcium ions into myocardial cells
e) Suppression of prostaglandin and thromboxane production
f) Venodilation and coronary artery vasodilation

A

[Y]Venodilation and coronary artery vasodilation

How well did you know this?
1
Not at all
2
3
4
5
Perfectly
12
Q

(Question ID: 134) A 75-year-old man is admitted to the hospital for treatment of congestive heart failure. He has a known history of diabetes, Alzheimer’s disease, and high- altitude pulmonary edema, in addition to a severe allergy to all sulfa derivatives. Which of the following drugs would be appropriate for diuresis in this patient?
a) Acetazolamide
b) Ethacrynic acid
c) Furosemide
d) Hydrochlorothiazide
e) Mannitol

A

[Y]Ethacrynic acid

How well did you know this?
1
Not at all
2
3
4
5
Perfectly
13
Q

(Question ID: 135) A 62-year-old man who has been in the hospital for the past 4 days begins decompensating in the middle of the night. When examined, his blood pressure is 60/30 mm Hg, pulse is 112/min, and temperature is 40.6.C (105 ٠F). His breathing is rapid and shallow, and his arms and legs are warm to the touch. The patient is immediately given a 2-L bolus of intravenous fluids, with no resulting change in blood pressure. He is then admitted to the intensive care unit for suspected septic shock. Which of the following drugs should be used initially to stabilize the patient’s vital signs?
a) Clonidine
b) Metoprolol
c) Norepinephrine
d) Phenoxybenzamine
e) Vancomycin

A

[Y]Norepinephrine

How well did you know this?
1
Not at all
2
3
4
5
Perfectly
14
Q

(Question ID: 136) A 35-year-old woman presents for management of hypertension. She has tried for the past 6 months to manage her hypertension with diet and exercise, without success, and says that she is willing to try medication. The patient asks about trying timolol because her mother had success with it in treating her own hypertension. What missing piece of medical history might affect the choice of medical therapy for this patient?
a) Acute dissecting aortic aneurysm
b) Asthma
c) Glaucoma
d) Hypertension
e) Mild congestive heart failure
f) Myocardial infarction

A

[Y]Asthma

How well did you know this?
1
Not at all
2
3
4
5
Perfectly
14
Q

(Question ID: 137) An 80-year-old woman with a history of acute myocardial infarction and congestive heart failure (CHF) presents to the emergency department with a chief complaint of fevers and lethargy. After a full work-up, a urinalysis is performed, yielding a specimen with a pH of 5.5 and increased sodium, chloride, potassium, and calcium concentrations. Her CHF is managed with lisinopril, metoprolol, and a diuretic. Which portion of the renal tubule is mostly likely affected by this patient’s diuretic?
a) Collecting tubule
b) Distal convoluted tubule
c) Multiple portions
d) Proximal convoluted tubule
e) Thick ascending limb

A

[Y]Thick ascending limb

How well did you know this?
1
Not at all
2
3
4
5
Perfectly
15
Q

(Question ID: 138) A 57-year-old man with a medical history significant for angina and hypertension presents because of worsening angina. He treats each episode with sublingual nitroglycerin but has noticed that over the past year the frequency of “episodes” has increased from once a week to daily. The patient is currently taking lisinopril. A note in the chart indicates that the patient is allergic to metoprolol, so a calcium channel blocker is added to the patient’s regimen; this medication has a clinical effect most similar
a) Amlodipine
b) Diltiazem
c) Nifedipine
d) Nimodipine
e) Verapamil

A

[Y]Verapamil

How well did you know this?
1
Not at all
2
3
4
5
Perfectly
16
Q

(Question ID: 139) A 67-year-old woman presents to the emergency department complaining of dizziness. During the interview, she experiences two episodes of near-syncope. Physical examination reveals palpitations and slight bradycardia. Her daughter, who accompanies her, states that the patient is taking a medication for ”heart troubles” but says she cannot remember its name. Results of ECG are shown in the image. Which of the following medications is this patient likely taking that could both explain her symptoms and produce the abnormalities shown in this ECG?
a) Adenosine
b) Bretylium
c) Propranolol
d) Quinidine
e) Verapamil

A

[Y]Quinidine

How well did you know this?
1
Not at all
2
3
4
5
Perfectly
17
Q

(Question ID: 140) Calcium channel blockers commonly are used to treat hypertension, ischemic heart disease, and arrhythmias. Calcium channel blockers can be categorized aS dihydroperidines (including nifedipine) and non-dihydroperidines (including diltiazem). The different clinical uses of these drugs depend on their contrasting modes of action. The effectiveness of diltiazem in treating cardiac arrhythmias can be attributed to what mechanism?
a) Increase in cardiac chronotropism
b) Increase in conduction velocity
c) Increase in vascular smooth muscle tone
d) Prolongation of the PR interval
e) Prolongation of the QT interval

A

[Y]Prolongation of the PR interval

How well did you know this?
1
Not at all
2
3
4
5
Perfectly
18
Q

(Question ID: 141) A 48-year-old woman with a history of hypercholesterolemia and smoking presents to the cardiologist complaining of activity-induced chest pain. She describes the pain aS a chest tightness that develops after walking several blocks and goes away after several minutes of rest. In addition to arranging further testing, the physician gives the patient a tablet and tells her to put it under her tongue when she has any similar episodes. At low doses, which of the following is the primary mechanism by which this medication alleviates the patient’s symptoms?
a) Decrease platelet activation in response to injury
b) Decreases afterload, decreases myocardial oxygen demands
c) Decreases plaque formation on coronary arteries
d) Decreases preload, decreases myocardial oxygen demands
e) Increases afterload and decreases myocardial oxygen demands
f) Increases preload, decreases myocardial oxygen demands

A

[Y]Decreases preload, decreases myocardial oxygen demands

How well did you know this?
1
Not at all
2
3
4
5
Perfectly
19
Q

(Question ID: 142) A 54-year-old contractor complains of anginal pain that occurs at rest. On examination, his blood pressure is 145/90 and his heart rate is 90. A treatment of angina that often decreases the heart rate and can prevent vasospastic angina attacks is
a) Diltiazem
b) Nifedipine
c) Nitroglycerin
d) Propranolol
e) Timolol

A

[Y]Diltiazem

How well did you know this?
1
Not at all
2
3
4
5
Perfectly
20
Q

(Question ID: 143) In a patient receiving digoxin for congestive heart failure, conditions that may facilitate the appearance of toxicity include
a) Hyperkalemia
b) Hypernatremia
c) Hypocalcemia
d) Hypomagnesemia
e) Hypophosphatemia

A

[Y]Hypomagnesemia

How well did you know this?
1
Not at all
2
3
4
5
Perfectly
21
Q

(Question ID: 144) A 29-year-old accountant has recurrent episodes of tachycardia that sometimes convert to sinus rhythm spontaneously but more often require medical treatment. A drug that is commonly given as an intravenous bolus for the purpose of converting AV nodal tachycardias to normal sinus rhythm is
a) Adenosine
b) Amiodarone
c) Lidocaine
d) Quinidine
e) Sotalol

A

[Y]Amiodarone

How well did you know this?
1
Not at all
2
3
4
5
Perfectly
22
Q

(Question ID: 145) A 67 year old man comes to the office because he is having difficulty maintaining an erection. He has a past medical history significant for hypertension, hyperlipidemia and diabetes mellitus type II. He notes that about once a month he will have chest pain on exertion that lasts for only a few minutes once he rests, but he denies any chest pain while at rest. He smokes one pack of cigarettes daily and drinks two to three beers a week. The physician prescribes him a medication for his erectile dysfunction but is concerned about interaction with his other medications. Taking which of the following cardiac medications would be contraindicated?
a) Hydrochlorothiazide
b) Lisinopril
c) Metiprolol
d) Nitrate
e) Simvastatine

A

[Y]Nitrate

How well did you know this?
1
Not at all
2
3
4
5
Perfectly
23
Q

(Question ID: 149) A 57-year-old man presents to his physician with periorbital swelling and edema of the lips and tongue. Two weeks prior to presentation, the patient was hospitalized with a ST segment elevation myocardial infarction. His blood pressure at the hospital was noted to be 174/103. The patient was discharged from the hospital and placed on several medications including aspirin; propanolol, lisinopril, isosorbide dinitrate, and hydrochlorothiazide. Which of the following is the most likely cause of this patient’s symptoms?
a) Aspirin
b) Hydrochlorothiazide
c) lsosorbide dinitrate
d) Lisinopril
e) Propranolol

A

[Y]Lisinopril

How well did you know this?
1
Not at all
2
3
4
5
Perfectly
24
Q

(Question ID: 150) A 69-year-old woman comes to the hospital with shortness of breath that is worst when she lays down in bed. Her past medical history is significant for hypertension, coronary artery disease; and diabetes mellitus. She also had an inferior wall myocardial infarction two years ago. Her last echocardiogram three months ago showed impaired systolic function with a left ventricular ejection fraction of 30%. Her blood pressure is 125/67, heart rate 73 beats per minute, and respiratory rate of 23 respirations per minute. Pulse oximetry shows oxygen saturation of 92% on room air. On examination she has crackles throughout her lung fields. Her cardiac examination reveals a normal S1 and S2 with no evidence of murmur. Chest x-ray shows bilateral pulmonary infiltrates consistent with pulmonary edema. Which of the following medications should be started to improve the patient’s shortness of breath?
a) Carvedilol
b) Furosemide
c) Lisinopril
d) Metoprolol
e) Spironolactone

A

[Y]Furosemide

How well did you know this?
1
Not at all
2
3
4
5
Perfectly
25
Q

(Question ID: 320) A 55-year-old woman with hypertension is to be treated with a vasodilator drug. Drugs X and Y have the same mechanism of action. Drug X in a dose of 5 mg produces the same decrease in blood pressure as 500 mg of drug Y. Which of the following statements best describes these results?
a) Drug Y is less efficacious than drug X
b) Drug X is about 100 times more potent than drug Y
c) Toxicity of drug X is less than that of drug Y
d) Drug X has a wider therapeutic window than drug Y
e) Drug X will have a shorter duration of action than drug Y because less of drug X is present over the time course of drug action

A

[Y]Drug X is about 100 times more potent than drug Y

How well did you know this?
1
Not at all
2
3
4
5
Perfectly
26
Q

(Question ID: 321) Sugammadex is a new drug that reverses the action of rocuronium and certain other skeletal muscle-relaxing agents. It appears to interact directly with the rocuronium molecule and not at all with the rocuronium receptor. Which of the following terms best describes sugammadex?
a) Chemical antagonist
b) Noncompetitive antagonist
c) Partial agonist
d) Pharmacologic antagonist
e) Physiologic antagonist

A

[Y]Chemical antagonist

How well did you know this?
1
Not at all
2
3
4
5
Perfectly
27
Q

(Question ID: 322) A study was carried out in isolated, perfused animal hearts. In the absence of other drugs, pindolol, a β-adrenoceptor ligand, caused an increase in heart rate. In the presence of highly effective β stimulants, however, pindolol caused a dose-dependent, reversible decrease in heart rate. Which of the following expressions best describes pindolol?
a) A chemical antagonist
b) An irreversible antagonist
c) A partial agonist
d) A physiologic antagonist
e) A spare receptor agonist

A

[Y]A partial agonist

How well did you know this?
1
Not at all
2
3
4
5
Perfectly
28
Q

(Question ID: 323) Verapamil and phenytoin are both eliminated from the body by metabolism in the liver. Verapamil has a clearance of 1.5 L/min, approximately equal to liver blood flow, whereas phenytoin has a clearance of 0.1 L/min. When these compounds are administered along with rifampin, a drug that markedly increases hepatic drug-metabolizing enzymes, which of the following is most likely?
a) The half-lives of both verapamil and phenytoin will be markedly increased
b) The clearance of both verapamil and phenytoin will be markedly decreased
c) The clearance of verapamil will be unchanged, whereas the clearance of phenytoin will be increased
d) The half-life of phenytoin will be unchanged, whereas the half-life of verapamil will be increased
e) The clearance of both drugs will be unchanged

A

[Y]The clearance of verapamil will be unchanged, whereas the clearance of phenytoin will be increased

How well did you know this?
1
Not at all
2
3
4
5
Perfectly
29
Q

(Question ID: 324) A 63-year-old woman in the intensive care unit requires an infusion of procainamide. Its half-life is 2 h. The infusion is begun at 9 AM. At 1 PM on the same day, a blood sample is taken; the drug concentration is found to be 3 mg/L. What is the probable steady-state drug concentration, for example, after 12 or more hours of infusion?
a) 3 mg/L
b) 4 mg/L
c) 6 mg/L
d) 9.9 mg/L
e) 15 mg/L

A

[Y]4 mg/L

How well did you know this?
1
Not at all
2
3
4
5
Perfectly
30
Q

(Question ID: 325) A 30-year-old man is brought to the emergency department in a deep coma. Respiration is severely depressed and he has pinpoint pupils. His friends state that he self-administered a large dose of morphine 6 h earlier. An immediate blood analysis shows a morphine blood level of 0.25 mg/L. Assuming that the Vd of morphine in this patient is 200 L and the half-life is 3 h, how much morphine did the patient inject 6 h earlier?
a) 25 mg
b) 50 mg
c) 100 mg
d) 200 mg

A

[Y]200 mg

How well did you know this?
1
Not at all
2
3
4
5
Perfectly
31
Q

(Question ID: 326) Gentamicin, an aminoglycoside antibiotic, is sometimes given in intermittent intravenous bolus doses of 100 mg 3 times a day to achieve target peak plasma concentrations of about 5 mg/L. Gentamicin’s clearance (normally 5.4 L/h/70 kg) is almost entirely by glomerular filtration. Your patient, however, is found to have a creatinine clearance one third of normal. What should your modified dosage regimen for this patient be?
a) 20 mg 3 times a day
b) 33 mg 3 times a day
c) 72 mg 3 times a day
d) 100 mg 2 times a day
e) 150 mg 2 times a day

A

[Y]33 mg 3 times a day

How well did you know this?
1
Not at all
2
3
4
5
Perfectly
32
Q

(Question ID: 327) A 3-year-old child has swallowed the contents of 2 bottles of a nasal decongestant whose primary ingredient is a potent, selective α-adrenoceptor agonist drug. Which of the following is a sign of α-receptor activation that may occur in this patient?
a) Bronchodilation
b) Cardiac acceleration (tachycardia)
c) Pupillary dilation (mydriasis)
d) Renin release from the kidneys
e) Vasodilation of the splanchnic vessels

A

[Y]Pupillary dilation (mydriasis)

How well did you know this?
1
Not at all
2
3
4
5
Perfectly
33
Q

(Question ID: 328) Full activation of the sympathetic nervous system, as in the fight-or-flight reaction, may occur during maximal exercise. Which of the following effects is likely to occur?
a) Bronchoconstriction
b) Increased intestinal motility
c) Decreased renal blood flow
d) Miosis
e) Decreased heart rate (bradycardia)

A

[Y]Decreased renal blood flow

How well did you know this?
1
Not at all
2
3
4
5
Perfectly
34
Q

(Question ID: 329) A 30-year-old woman undergoes abdominal surgery. In spite of minimal tissue damage, complete ileus (absence of bowel motility) follows, and she complains of severe bloating. She also finds it difficult to urinate. Mild cholinomimetic stimulation with bethanechol or neostigmine is often effective in relieving these complications of surgery. Neostigmine and bethanechol in moderate doses have significantly different effects on which one of the following?
a) Gastric secretion
b) Neuromuscular end plate
c) Salivary glands
d) Sweat glands
e) Ureteral tone

A

[Y]Neuromuscular end plate

How well did you know this?
1
Not at all
2
3
4
5
Perfectly
35
Q

(Question ID: 330) A 63-year-old women has been treated for myasthenia gravis for several years. She reports to the emergency department complaining of recent onset of weakness of her hands, diplopia, and difficulty swallowing. She may be suffering from a change in response to her myasthenia therapy, that is, a cholinergic or a myasthenic crisis. Which of the following is the best drug for distinguishing between myasthenic crisis (insufficient therapy) and cholinergic crisis (excessive therapy)?
a) Atropine
b) Edrophonium
c) Physostigmine
d) Pralidoxime
e) Pyridostigmine

A

[Y]Edrophonium

How well did you know this?
1
Not at all
2
3
4
5
Perfectly
36
Q

(Question ID: 331) A 63-year-old man has just been diagnosed with dysautonomia (chronic idiopathic autonomic insufficiency). You are considering different therapies for his disease. Pyridostigmine and neostigmine may cause which one of the following?
a) Bronchodilation
b) Cycloplegia
c) Diarrhea
d) Irreversible inhibition of acetylcholinesterase
e) Reduced gastric acid secretion

A

[Y]Diarrhea

How well did you know this?
1
Not at all
2
3
4
5
Perfectly
37
Q

(Question ID: 332) Actions and clinical uses of muscarinic cholinoceptor agonists include which one of the following?
a) Bronchodilation (asthma)
b) Improved aqueous humor drainage (glaucoma)
c) Decreased gastrointestinal motility (diarrhea)
d) Decreased neuromuscular transmission and relaxation of skeletal muscle (during surgical anesthesia)
e) Increased sweating (fever)

A

[Y]Improved aqueous humor drainage (glaucoma

How well did you know this?
1
Not at all
2
3
4
5
Perfectly
38
Q

(Question ID: 333) A 3-year-old child is admitted after taking a drug from her parents’ medicine cabinet. The signs suggest that the drug is an indirect-acting cholinomimetic with little or no CNS effect and a duration of action of about 2–4 h. Which of the following is the most likely cause of these effects?
a) Acetylcholine
b) Bethanechol
c) Neostigmine
d) Physostigmine
e) Pilocarpine

A

[Y]Neostigmine

How well did you know this?
1
Not at all
2
3
4
5
Perfectly
39
Q

(Question ID: 334) Which of the following is an expected effect of a therapeutic dose of an antimuscarinic drug?
a) Decreased cAMP (cyclic adenosine monophosphate) in cardiac muscle
b) Decreased DAG (diacylglycerol) in salivary gland tissue
c) Increased IP3 (inositol trisphosphate) in intestinal smooth muscle
d) Increased potassium efflux from smooth muscle
e) Increased sodium influx into the skeletal muscle end plate

A

[Y]Decreased DAG (diacylglycerol) in salivary gland tissue

How well did you know this?
1
Not at all
2
3
4
5
Perfectly
40
Q

(Question ID: 335) A 65-year-old woman with long-standing diabetes mellitus is admitted to the ward from the emergency department, and you wish to examine her retinas for possible changes. Which of the following drugs is a good choice when pupillary dilation—but not cycloplegia—is desired?
a) Isoproterenol
b) Norepinephrine
c) Phenylephrine
d) Pilocarpine
e) Tropicamide

A

[Y]Phenylephrine

How well did you know this?
1
Not at all
2
3
4
5
Perfectly
41
Q

(Question ID: 336) A 30-year-old man is admitted to the emergency department after taking a suicidal overdose of reserpine. His blood pressure is 50/0 mm Hg and heart rate is 40 bpm. Which of the following would be the most effective cardiovascular stimulant?
a) Amphetamine
b) Clonidine
c) Cocaine
d) Norepinephrine
e) Tyramine

A

[Y]Norepinephrine

How well did you know this?
1
Not at all
2
3
4
5
Perfectly
42
Q

(Question ID: 337) A 56-year-old man has hypertension and an enlarged prostate, which biopsy shows to be benign prostatic hyperplasia. He complains of urinary retention. Which of the following drugs would be the most appropriate initial therapy?
a) Albuterol
b) Atenolol
c) Metoprolol
d) Prazosin
e) Timolol

A

[Y]Prazosin

How well did you know this?
1
Not at all
2
3
4
5
Perfectly
43
Q

(Question ID: 338) Which of the following best describes the mechanism of action of benzodiazepines?
a) Activate GABA-B receptors in the spinal cord
b) Block glutamate receptors in hierarchical neuronal pathways in the brain
c) Increase frequency of opening of chloride ion channels coupled to GABA-A receptors
d) Inhibit GABA transaminase to increase brain levels of GABA
e) Stimulate release of GABA from nerve endings in the brain

A

[Y]Increase frequency of opening of chloride ion channels coupled to GABA-A receptors

How well did you know this?
1
Not at all
2
3
4
5
Perfectly
44
Q

(Question ID: 339) Which drug used in the maintenance treatment of patients with tonic-clonic or partial seizure states increases the hepatic metabolism of many drugs including both warfarin and phenytoin?
a) Buspirone
b) Chlordiazepoxide
c) Eszopiclone
d) Phenobarbital
e) Triazolam

A

[Y]Phenobarbital

How well did you know this?
1
Not at all
2
3
4
5
Perfectly
45
Q

(Question ID: 340) A young male patient suffers from a seizure disorder characterized by tonic rigidity of the extremities followed in 15–30 s of tremor progressing to massive jerking of the body. This clonic phase lasts for 1 or 2 min, leaving the patient in a stuporous state. Of the following drugs, which is most suitable for longterm management of this patient?
a) Carbamazepine
b) Clonazepam
c) Ethosuximide
d) Felbamate
e) Tiagabine

A

[Y]Carbamazepine

How well did you know this?
1
Not at all
2
3
4
5
Perfectly
46
Q

(Question ID: 341) A 30-year-old male patient is on drug therapy for a psychiatric problem. He complains that he feels “flat” and that he gets confused at times. He has been gaining weight and has lost his sex drive. As he moves his hands, you notice a slight tremor. He tells you that since he has been on medication he is always thirsty and frequently has to urinate. The drug he is most likely to be taking is
a) Carbamazepine
b) Clozapine
c) Fluphenazine
d) Lithium
e) Valproic acid

A

[Y]Lithium

How well did you know this?
1
Not at all
2
3
4
5
Perfectly
47
Q

(Question ID: 342) Which of the following drugs has a high affinity for 5-HT2 receptors in the brain, does not cause extrapyramidal dysfunction or hematotoxicity, and is reported to increase the risk of significant QT prolongation?
a) Clozapine
b) Haloperidol
c) Olanzapine
d) Ziprasidone

A

[Y]Ziprasidone

How well did you know this?
1
Not at all
2
3
4
5
Perfectly
48
Q

(Question ID: 343) Which of the following drugs is established to be both effective and safe to use in a pregnant patient suffering from bipolar disorder?
a) Carbamazepine
b) Lithium
c) Quetiapine
d) Valproic acid

A

[Y]Quetiapine

49
Q

(Question ID: 344) A 70-year-old woman is admitted to the emergency department because of a “fainting spell” at home. She appears to have suffered no trauma from her fall, but her blood pressure is 120/60 when lying down and 60/20 when she sits up. Neurologic examination and an ECG are within normal limits when she is lying down. Questioning reveals that she has recently started taking “water pills” (diuretics) for a heart condition. Which of the following drugs is the most likely cause of her fainting spell?
a) Acetazolamide
b) Amiloride
c) Furosemide
d) Spironolactone

A

[Y]Furosemide

50
Q

(Question ID: 345) A 64-year-old man is brought to the emergency department complaining of crushing chest pain radiating to his left arm. He is admitted, stabilized, and treated for an acute myocardial infarction. Later, he developed ventricular tachycardia and is treated with an antiarrhythmic. After a week of antiarrhythmic treatment, he began having difficulty breathing. A chest X-ray reveals pulmonary fibrosis. Which antiarrhythmic was he taking?
a) Amiodarone
b) Digoxin
c) Lidocaine
d) Procainamide
e) Verapamil

A

[Y]Amiodarone

51
Q

(Question ID: 346) A 67-year-old man is hospitalized recovering from a left wall myocardial infarction. He begins to show signs of fluid retention. His doctors want to start a drug regimen for congestive heart failure, including either an ACE inhibitor or an angiotensin receptor blocker (ARB). ACE inhibitors and ARBs treat hypertension in a similar fashion and have similar side effects. Which of the following is a side effect of ACE inhibitors only?
a) Dizziness
b) Dry cough
c) Erectile dysfunction
d) Hypotension
e) Tinnitus

A

[Y]Dry cough

52
Q

(Question ID: 347) A 62-year-old man has developed worsening hypertension despite therapy. His physician wants to prescribe an additional medication that will dilate his blood vessels to help lower his blood pressure. Which of the following is a calcium channel blocker that works primarily on vascular smooth muscle?
a) Amlodipine
b) Diltiazem
c) Losartan
d) Nitroprusside

A

[Y]Amlodipine

53
Q

(Question ID: 348) A 51-year-old man recently started treatment for an arrhythmia. He now presents with what appears to be a sunburn on his face, although he insists he has spent very little time in the sun. Which of the following drugs is he most likely taking?
a) Amiodarone
b) Lidocaine
c) Procainamide
d) Timolol
e) Verapamil

A

[Y]Procainamide

54
Q

(Question ID: 349) A 19-year-old woman is 24 weeks pregnant. She has received no prenatal care. She presents to the emergency department complaining of an intermittent headache and fatigue during her pregnancy. Her blood pressure has been at least 150/110 mm Hg. What is the most appropriate treatment of this patient?
a) Hydralazine
b) Prazosin
c) Sodium nitroprusside
d) Labetalol

A

[Y]Hydralazine

55
Q

(Question ID: 350) A 62-year-old woman with a history of diabetes mellitus, hypertension, and bilateral lower extremity edema presents to her primary care physician. Physical examination reveals pitting edema of both lower extremities from the ankles to the knees. She was placed on furosemide. The mechanism of action of this agent is in which of the following locations?
a) Ascending limb of the loop of Henle
b) Collecting duct
c) Distal tubule
d) Proximal tubule

A

[Y]Ascending limb of the loop of Henle

56
Q

(Question ID: 351) A 53-year-old female presents to the emergency department in acute distress from a rapid heart rate and chest pain. She is placed on continuous ECG monitoring and an arrhythmia of supraventricular tachycardia (SVT) is diagnosed. A 6-mg dose of IV adenosine is given and the patient converts back to a normal sinus rhythm. Where does adenosine act on the heart and what is the mechanism of action of adenosine?
a) AV node, increases efflux of K
b) AV node, increases influx of K
c) SA node, decreases intracellular Ca
d) SA node, increases efflux of K
e) SA node, increases influx of K

A

[Y]AV node, increases efflux of K

57
Q

(Question ID: 352) An 81-year-old female presents to the emergency department with a racing heartbeat. She is subsequently placed on continuous ECG monitor and irregular rhythm is seen. The diagnosis of atrial fibrillation is made. She is started on an amiodarone drip. What side effect should the physician be aware of when starting amiodarone?
a) Blurry yellow vision
b) Impotence
c) Lupus-like syndrome
d) Prolongation of AV refractory period
e) Pulmonary fibrosis

A

[Y]Pulmonary fibrosis

58
Q

(Question ID: 353) A 51-year-old man presents to the urology clinic with difficulty starting and stopping his stream during urination. The symptoms have been worsening over the past year. He also reports that he does not feel he empties his bladder completely. He is started on tamsulosin for benign prostatic hyperplasia. What is a common side effect of tamsulosin?
a) Cough
b) Hemorrhagic cystitis
c) Hypercoagulable state
d) Impaired blue-green vision
e) Orthostatic hypotension

A

[Y]Orthostatic hypotension

59
Q

(Question ID: 354) A 57-year-old man presents to his primary care physician for follow-up. He is found to have a blood pressure of 150/100 mm Hg on three successive occasions. He has begun on an antihypertensive agent. After taking the first dose of the medication, he goes out to play golf and collapses on the golf course. Which one of the following drugs may be responsible for his symptoms?
a) Atenolol
b) Hydrochlorothiazide
c) Metoprolol
d) Prazosin
e) Verapamil

A

[Y]Prazosin

60
Q

(Question ID: 355) A 32-year-old pregnant female presents to her obstetrician for a well-baby evaluation. Her blood pressure readings over the past two visits have both been 165/100 mm Hg. The physician suggests that antihypertensive medication is needed in this patient. Chest X-ray is obtained and shows no evidence of infiltrate or effusion. No mass lesions are noted, and the costophrenic angles are sharp. What pharmacologic agent is most effective and safest for this patient?
a) Furosemide
b) Hydrochlorothiazide
c) Lisinopril
d) Losartan
e) Methyldopa

A

[Y]Methyldopa

61
Q

(Question ID: 356) A 72-year-old man is brought to the emergency department after he passed out at home. Electrocardiography reveals an AV nodal–based arrhythmia. What drug would be most effective at interrupting the AV transmission and breaking the arrhythmic cycle at the AV node?
a) Adenosine
b) Flecainide
c) Lidocaine
d) Phenytoin
e) Quinidine

A

[Y]Adenosine

62
Q

(Question ID: 357) A physician decides to place a patient on a calcium channel blocker for treatment of her angina. Calcium channel blockers can relax the smooth muscle of blood vessels and can also have various effects on cardiac contractility, conduction, and heart rate. Which of the following calcium channel blockers would be most effective in reducing heart rate and contractility?
a) Dihydropyridine
b) Diltiazem
c) Nifedipine
d) Nimodipine
e) Verapamil

A

[Y]Verapamil

63
Q

(Question ID: 358) A patient was given a 200 mg dose of a drug IV, and 100 mg was eliminated during the first two hours. If the drug follows first-order elimination kinetics, how much of the drug will remain 6 hours after its administration?
a) None
b) 25 mg
c) 50 mg
d) 75 mg
e) 100 mg

A

[Y]25 mg

64
Q

(Question ID: 359) Drugs that are administered IV are:
a) Rapidly absorbed
b) Subject to first-pass metabolism
c) 100% bioavailable
d) Rapidly excreted by the kidneys
e) Rapidly metabolized by the liver

A

[Y]100% bioavailable

65
Q

(Question ID: 360) With IV infusion, a drug reaches 50% of its final steady state in 6 hours. The elimination half-life of the drug must be approximately:
a) 2 h
b) 6 h
c) 12 h
d) 24 h
e) 30 h

A

[Y]6 h

66
Q

(Question ID: 361) A 5-year-old child becomes ill while visiting relatives who have a farm in Arkansas. His symptoms include severe abdominal cramps with vomiting and diarrhea and profuse lacrimation and salivation. Pupillary constriction is marked. The most likely cause is exposure to
a) herbicides
b) antifreeze
c) lead-based paint
d) insecticides
e) rat poison

A

[Y]insecticides

67
Q

(Question ID: 362) The activation of muscarinic receptors in bronchiolar smooth muscle is associated with
a) activation of adenylyl cyclase
b) decrease in cAMP formation mediated by G-proteins
c) [Y]increase in IP 3 and DAG inhibition of protein kinase C
d) opening of Na+/K+ cation channels

A

[Y]increase in IP 3 and DAG inhibition of protein kinase C

68
Q

(Question ID: 363) Prior to an eye exam a patient is given a drug that causes mydriasis but has no effect on accommodation. What is the most likely identity of this drug?
a) mecamylamine
b) neostigmine
c) pilocarpine
d) phenylephrine
e) tropicamide

A

[Y]phenylephrine

69
Q

(Question ID: 364) Following pretreatment with a muscarinic receptor blocking agent, the IV administration of norepinephrine is likely to result in
a) ↑ HR and ↑ BP
b) ↑ HR and ↓ BP
c) ↓ HR and ↓ BP
d) ↓ HR and ↑ BP
e) no effect on HR, but ↑ BP

A

[Y]↑ HR and ↑ BP

70
Q

(Question ID: 365) A colleague with myasthenia gravis wants you to assist him to the ER because he is experiencing muscle weakness and has found it difficult to titrate his drug dosage because he has had the “flu.” You note that he has a slight temperature, shallow respirations, and a gray-blue skin pallor. What would be the most appropriate drug to give to your colleague at this time?
a) Albuterol
b) Edrophonium
c) Propranolol
d) Physostigmine
e) Scopolamine

A

[Y]Edrophonium

71
Q

(Question ID: 366) A 40-year-old man was given a drug that binds to a subunit of the GABA-A receptor. When used at a high dose, the drug can open Cl– channels independent of GABA. What drug was the man given?
a) Diazepam
b) Ethanol
c) [Y]Phenobarbital
d) Baclofen
e) Dronabinol

A

[Y]Phenobarbital

72
Q

(Question ID: 367) A patient comes to the ER with a painful stab wound. The ER resident administers pentazocine for the pain. Soon after administration the patient experiences sweating, restlessness, and an increase in pain sensations. What is the most likely explanation for his symptoms?
a) The patient is probably tolerant to pentazocine
b) The patient is a heroin addict
c) Pentazocine is an ineffective analgesic
d) Pentazocine was used at the wrong dose
e) Pentazocine doesn’t cross the blood-brain barrier

A

[Y]The patient is a heroin addict

73
Q

(Question ID: 368) What Is the rationale for combining levodopa with carbidopa?
a) Carbidopa stimulates dopamine receptors
b) Carbidopa increases levodopa entry into the CNS by inhibiting peripheral dopa decarboxylase
c) Carbidopa enhances levodopa absorption
d) Carbidopa enhances the peripheral conversion of levodopa to dopamine
e) Carbidopa blocks peripheral COMT

A

[Y]Carbidopa increases levodopa entry into the CNS by inhibiting peripheral dopa decarboxylase

74
Q

(Question ID: 369) A hospital nurse is taking imipramine for a phobic anxiety disorder, and her patient is being treated with chlorpromazine for a psychotic disorder. Which of the following adverse effects is likely to occur in both of these individuals?
a) Excessive salivation
b) Pupillary constriction
c) Orthostatic hypotension
d) Seizure threshold
e) Weight loss

A

[Y]Orthostatic hypotension

75
Q

(Question ID: 370) Which one of the following is characteristic of both phenytoin and carbamazepine?
a) Inhibition of hepatic cytochrome P450
b) First-order elimination at high therapeutic doses
c) Enhances the effects of oral contraceptives
d) Safe to use in pregnancy
e) Prevent sodium influx through fast sodium channels

A

[Y]Prevent sodium influx through fast sodium channels

76
Q

(Question ID: 371) Midazolam is an effective anesthetic because it acts by
a) increasing functional activity at GABA-B receptors
b) enhancing the actions of dopamine
c) blocking the NMDA glutamate receptor subtype
d) acting as a partial agonist at 5HT receptors
e) facilitating GABA-mediated increases in chloride ion conductance

A

[Y]facilitating GABA-mediated increases in chloride ion conductance

77
Q

(Question ID: 372) Tricyclic antidepressants
a) have anticonvulsant activity
b) should not be used in patients with glaucoma
c) may increase oral absorption of levodopa
d) are sometimes used as antiarrhythmics

A

[Y]should not be used in patients with glaucoma

78
Q

(Question ID: 373) Which side effect is associated with spironolactone?
a) Alkalosis
b) Hirsutism
c) Hyperkalemia
d) Hypercalcemia
e) Hyperglycemia

A

[Y]Hyperkalemia

79
Q

(Question ID: 374) Lidocaine is an effective antiarrhythmic because it
a) suppresses excitability in hypoxic areas of the heart
b) prolongs the QT interval
c) prolongs the PR interval
d) depresses the slope of phase 0 in slow response tissues
e) acts on inhibitory G-protein coupled receptors

A

[Y]suppresses excitability in hypoxic areas of the heart

80
Q

(Question ID: 375) A patient has a genetic polymorphism such that they cannot rapidly metabolize drugs by acetylation. You would be most concerned about this polymorphism if the patient was taking which drug?
a) Sotalol
b) Clonidine
c) Nitroglycerin
d) [Y]Hydralazine
e) Prazosin

A

[Y]Hydralazine

81
Q

(Question ID: 376) A patient with hypertension also suffers from essential tremor. Optimal treatment of the patient should include management with
a) prazosin
b) clonidine
c) metoprolol
d) lidocaine
e) propranolol

A

[Y]propranolol

82
Q

Question ID: 377) Selective β-1 blockers are preferred over nonselective beta blockers in some patients because they
a) cause less cardiodepression
b) are less likely to cause bronchoconstriction
c) are more effective for migraine prophylaxis
d) are more effective as an antiarrhythmics
e) have greater prophylactic value post-MI

A

[Y]are less likely to cause bronchoconstriction

83
Q

(Question ID: 378) A 75-year-old patient suffering from congestive heart failure accidentally ingests a toxic dose of digoxin. Clinical consequences due to the toxic effects of cardiac glycosides are likely to include
a) seizures
b) hypercalcemia
c) bicarbonaturia
d) intermittent claudication
e) visual disturbances

A

[Y]visual disturbances

84
Q

(Question ID: 379) A new diuretic is being studied in human volunteers. Compared with placebo, the new drug increases urine volume, increases urinary Ca2+, increases plasma pH, and decreases serum K+. If this new drug has a similar mechanism of action to an established diuretic, it probably
a) blocks the NaCl cotransporter in the DCT
b) blocks aldosterone receptors in the CT
c) inhibits carbonic anhydrase in the PCT
d) inhibits the Na+/K+/2Cl– cotransporter in the TAL
e) acts as an osmotic diuretic

A

[Y]inhibits the Na+/K+/2Cl– cotransporter in the TAL

85
Q

(Question ID: 380) Outpatient prophylaxis of a patient with an SVT is best accomplished with the administration of
a) adenosine
b) diltiazem
c) esmolol
d) lidocaine
e) mexilitene

A

[Y]diltiazem

86
Q

(Question ID: 381) Enhancement of the effects of bradykinin is most likely to occur with drugs like
a) clonidine
b) diazoxide
c) lisinopril
d) losartan
e) propranolol

A

[Y]lisinopril

87
Q

(Question ID: 382) Which one of the following drugs is most likely to cause symptoms of severe depressive disorder when used in the treatment of hypertensive patients?
a) Captopril
b) Hydrochlorothiazide
c) Prazosin
d) Nifedipine
e) Reserpine

A

[Y]Reserpine

88
Q

(Question ID: 383) Which one of the following drugs is most likely to block K+ channels in the heart responsible for cardiac repolarization, and also blocks calcium channels in the AV node?
a) Amiodarone
b) Quinidine
c) Lidocaine
d) Sotalol
e) Verapamil

A

[Y]Amiodarone

89
Q

(Question ID: 384) Both dobutamine and inamrinone increase cardiac contractility by
a) activation of adenylyl cyclase
b) inactivation of Na channels
c) inhibition of Na+/K+-ATPase
d) increasing cAMP
e) activation of Na/Cl cotransporter

A

[Y]increasing cAMP

90
Q

(Question ID: 385) A patient with a supraventricular tachycardia has an atrial rate of 280/min with a ventricular rate of 140/min via a 2:1 AV nodal transmission. After treatment with a drug, the atrial rate slowed to 180/min, but the ventricular rate increased to 180/min! Which of the following drugs was most likely to have been given to this patient?
a) Adenosine
b) Digoxin
c) Esmolol
d) Quinidine
e) Verapamil

A

[Y]Quinidine

91
Q

(Question ID: 386) A 67-year-old man injures his shoulder in an ATV accident. Over-the-counter ibuprofen is unable to control the pain satisfactorily. The patient asks about glucocorticoid injections, so his doctor begins to explain the myriad effects of glucocorticoids in the body. Which of the following glucocorticoid actions would be most desirable in this patient?
a) Antiemetic
b) Decreasing synthesis of HGF, a growth factor
c) Decreasing translocation of GLUT4 receptors to the cell membrane
d) Increasing synthesis of IkB
e) Stimulating gluconeogenesis

A

[Y]Increasing synthesis of IkB

92
Q

(Question ID: 387) A 37-year-old kidney transplant recipient presents to her primary care physician for follow-up. Among other immunosuppressant drugs, she has been taking daily prednisone for the past 2 months since her transplant. With only a few doses of prednisone left, she gets snowed into her house and cannot refill her prescription (but she has enough of the other medications to last a few more weeks). If she runs out of prednisone and cannot get it refilled, what is she most at risk for developing?
a) Cardiovascular collapse (adrenal crisis)
b) Osteoporosis
c) Increased risk of infection
d) Insomnia (short-term oral/parenteral)
e) Nausea/vomiting (short-term oral/parenteral)

A

[Y]Osteoporosis

93
Q

(Question ID: 388) A 72-year-old woman with myasthenia gravis is brought to the emergency department with decreased responsiveness. She has a history of diabetes, hypertension, Alzheimer’s disease, and stroke. Physical examination reveals significant abdominal tenderness with guarding and peritoneal signs. Which of the following is a medication that should be discontinued immediately in this patient?
a) Insulin
b) Neostigmine
c) Nifedipine
d) Rivastigmine
e) Tacrine

A

[Y]Neostigmine

94
Q

(Question ID: 389) A 58-year-old man with depression who takes a monoamine oxidase inhibitor has a love for aged cheese and wine. His depression is well controlled in this regimen. He must be alert to which of the following significant events?
a) Diarrhea
b) Hypertensive
c) headaches
d) Lacrimation
e) Sweating
f) Tremors

A

[Y]Hypertensive

95
Q

(Question ID: 390) Five patients with sexual dysfunction present to their primary care physician for evaluation and treatment. Which of the following would most likely benefit from treatment with this medication yohimbine?
a) A 19-year-old man with sexual arousal difficulties
b) A 29-year-old man with erectile and ejaculatory dysfunction
c) A 47-year-old man with erectile dysfunction and hypertension
d) A 57-year-old man with erectile dysfunction and diabetes mellitus
e) An 80-year-old man with erectile dysfunction and myocardial infarction

A

[Y]A 19-year-old man with sexual arousal difficulties

96
Q

(Question ID: 391) A 29-year-old man with a family history of heart disease presents to his primary care physician for a routine checkup. A lipid profile on a blood draw reveals high LDL and low HDL. One way to decrease the amount of LDL in the blood is to hinder the liver’s ability for de novo cholesterol synthesis. Which of the following drugs blocks de novo cholesterol synthesis in hepatocytes?
a) Cholestyramine
b) Colesevelam
c) Colestipol
d) Ezetimibe
e) Rosuvastatin

A

) [Y]Rosuvastatin

97
Q

(Question ID: 392) A 37-year-old woman with hyperlipidemia is taking a drug to lower her triglyceride and blood cholesterol levels. She is considering stopping her therapy, however, because of a red, itchy rash on her face and neck that occurs following some doses. Which drug is she taking?
a) Atorvastatin
b) Fenofibrate
c) Gemfibrozil
d) Nicotinic acid
e) Omacor

A

[Y]Nicotinic acid

98
Q

(Question ID: 393) A 41-year-old man with end-stage HIV disease has lost approximately 50 lb from his baseline weight of 160 lb. He now weighs in the fifth percentile for his age group. He is given dronabinol to achieve a change in this area. The most plausible mechanism of action for this medication is
a) Appetite stimulant Induction of chemotactic centers of the brain
b) Stimulation of epidermal growth factor
c) Stimulation of keratinocyte growth factor
d) Stimulation of growth hormone

A

[Y]Appetite stimulant Induction of chemotactic centers of the brain

99
Q

(Question ID: 394) A 63-year-old woman falls at home and fractures her wrist. She has a 40 pack-year history of smoking. Her doctor recommends a DXA scan, which reveals a very low bone density and prescribes alendronate. How will alendronate help this patient?
a) Enhancing GI calcium absorption
b) Inhibiting calcium excretion in the kidneys
c) Inhibiting osteoclasts
d) Providing the starting material for bone mineralization
e) Stimulating osteoblasts

A

[Y]Inhibiting osteoclasts

100
Q

(Question ID: 395) A 33-year-old woman presents to her primary care physician with tachycardia, heat intolerance, tremor, and unintentional weight loss. A thyroid scan shows multiple regions of thyroid taking up excess iodine. She is prescribed with a drug that will decrease synthesis of thyroid hormones and decrease the peripheral conversion of T 4 to T3. Which drug is this?
a) Lanreotide
b) Levothyroxine
c) Methimazole
d) Octreotide
e) Propylthiouracil

A

[Y]Propylthiouracil

101
Q

(Question ID: 396) A 27-year-old man was recently diagnosed with Type-2 diabetes mellitus and placed on a medication. As he was drinking with his friends, he became violently ill. What medication is he most likely taking?
a) Acarbose
b) Glyburide
c) Metformin
d) Pioglitazone
e) Tolbutamide

A

[Y]Tolbutamide

102
Q

(Question ID: 397) A 43-year-old woman with Type-2 diabetes has been taking insulin with meals as well as metformin. Her blood glucose remains poorly controlled. Her doctor prescribes an additional drug, which is an analog of an endogenous peptide that inhibits glucagon secretion. What is the most likely medication this patient is taking?
a) Exenatide
b) Glipizide
c) Miglitol
d) Pramlintide
e) Rosiglitazone

A

[Y]Pramlintide

103
Q

(Question ID: 398) A 57-year-old man with a 40 pack-year history of smoking develops small cell lung cancer. His serum sodium is 121 mEq/L and his urine has a specific gravity of 1.030. Which of the following is a tetracycline antibiotic that can also be used in the treatment of SIADH?
a) Demeclocycline
b) Doxycycline
c) Minocycline
d) Tetracycline
e) Tigecycline

A

[Y]Demeclocycline

104
Q

(Question ID: 399) A young married couple has been unable to conceive after 3 years of unprotected intercourse. The husband’s sperm count is normal. They both agree that they would like the wife to undergo fertility treatment. Which of the following drugs has the ability to either stimulate or inhibit ovulation depending only on the dosing schedule?
a) Bromocriptine
b) Clomiphene
c) Estrone sulfate hCG
d) Leuprolide

A

[Y]Leuprolide

105
Q

(Question ID: 400) A 59-year-old man with long-standing hypertension presents to his primary care physician for a follow-up visit and guidance. His blood pressure is 160/80 mm Hg. His current medications include a calcium channel blocker. He is now switched to losartan. Which of the following properties of this medication can limit its efficacy for this patient?
a) Extensive hepatic first-pass metabolism
b) Drug interaction with antacids
c) Oral administration
d) Renal excretion
e) Requirement to be taken on a full stomach

A

[Y]Extensive hepatic first-pass metabolism

106
Q

(Question ID: 401) A 52-year-old man is prescribed with furosemide by his physician to help treat his hypertension. His physician further advises him to avoid NSAIDs for pain because they may antagonize the effects of furosemide. Which of the following describes how NSAIDs may interfere with the effects of furosemide?
a) Afferent arteriole vasoconstriction
b) Efferent arteriole vasoconstriction
c) Increase tubular sodium reabsorption
d) Stimulation of the ADH receptor
e) The physician is mistaken; pain management with NSAIDs has no bearing on furosemide use

A

[Y]Afferent arteriole vasoconstriction

107
Q

(Question ID: 402) A 19-year-old woman presents to her primary care physician for evaluation of elevated triglycerides. Her mother, father, sister, and brother all have the same problem. Her serum cholesterol and triglyceride levels are normal. What is the most appropriate initial treatment for this patient?
a) Dietary modification to include increased protein and fats
b) Exercise regimen approximately 5 d/wk
c) HMG-CoA reductase inhibitor 5 d/wk
d) Introduction of a statin drug 5 d/wk
e) Watchful waiting and follow-up in 1 year

A

[Y]Exercise regimen approximately 5 d/wk

108
Q

(Question ID: 403) A 45-year-old man has high serum LDL and low serum HDL. He presents to his primary care physician for treatment guidance. In addition to exercise and diet modification, which of the following agents would provide him with the most efficacious improvement of serum LDL and HDL levels?
a) Cholestyramine
b) Fluvastatin
c) Lovastatin
d) Simvastatin

A

[Y]Simvastatin

109
Q

(Question ID: 404) A 39-year-old man with insulin-dependent diabetes mellitus is brought to the emergency department after collapsing in a shopping mall. His blood sugar is 589 mg/dL. Which of the following preparations would have the least minimal effect on his blood sugar levels?
a) Insulin aspart
b) Insulin glargine
c) Insulin lispro
d) Regular insulin

A

[Y]Insulin glargine

110
Q

(Question ID: 405) A 32-year-old woman with endometriosis of the uterus, pelvic sidewall, and bladder has chronic unrelenting pelvic pain. She is prescribed with danazol. Over the next few weeks, she experiences a 40% improvement in her symptoms. What physiologic changes are likely to be found in this patient as a result of this medication?
a) Decreased aromatase
b) Decreased hair growth
c) Decreased libido
d) Decreased FSH
e) Decreased testosterone

A

[Y]Decreased FSH

111
Q

(Question ID: 406) An 18-year-old male high school baseball player is interested in improving his hitting ability. He receives twice weekly injections of anabolic steroids. Over the next year, which of the following effects would be likely observed in this patient?
a) Depression
b) Hepatic hemangioma
c) Premature epiphyseal long bone closing
d) Testicular hyperplasia
e) Tubercle development

A

[Y]Premature epiphyseal long bone closing

112
Q

(Question ID: 407) A 66-year-old woman with a history of Cushing’s disease treated with oral glucocorticoids presents to her primary care physician for follow-up. She was recently hospitalized for a hip fracture following a fall. What is the most likely mechanism for hip fracture in this patient?
a) Increased intestinal calcium absorption
b) Increased sex hormone synthesis
c) Inhibition of bone formation
d) Osteoarthritis
e) Osteochondroma

A

[Y]Inhibition of bone formation

113
Q

(Question ID: 408) A 48-year-old postmenopausal woman undergoes a hysterectomy prophylactically because of a strong family history of endometrial cancer. After her procedure, she complained of difficulty moving her bowels. Physical exam revealed decreased bowel sounds in all four quadrants. A white blood cell count showed 9,000 cells per microliter. The physician gives her a drug to stimulate her intestines until normal function resumes. Which of the following is most likely the drug given?
a) Benztropine
b) Bethanechol
c) Epinephrine
d) Methscopolamine
e) Oxybutynin

A

[Y]Bethanechol

114
Q

(Question ID: 409) A 37-year-old woman with diabetes is brought to the emergency department unresponsive by her husband. She recently started taking a new medication to control her blood sugar, but her husband could not remember the name of it. Her blood sugar is 45 mg/dL. Which of the following diabetes medications is most likely for her condition?
a) Acarbose
b) Glipizide
c) Metformin
d) Pramlintide
e) Sitagliptin

A

[Y]Glipizide

115
Q

(Question ID: 410) A cell within the body that secretes a chemical that acts locally on cells in their immediate environment and does not enter the blood stream describes which of the following?
a) Epinephrine
b) Histamine
c) Histamine
d) Thyroxine
e) Testosterone

A

[Y]Histamine

116
Q

(Question ID: 411) A 28-year-old man who is obese is found to have a hemoglobin A1c of 9.5%. He has been unable to adequately control his blood sugar with diet and exercise alone. His physician wishes to prescribe an insulin product to help control his blood sugar level. Which of the following is the longest acting to provide this patient a low, baseline insulin dose that will last throughout the day?
a) Insulin aspart
b) Insulin glargine
c) Insulin lispro
d) Lente insulin
e) NPH insulin

A

[Y]Insulin glargine

117
Q

(Question ID: 412) A 43-year-old man with dyslipidemia comes to the clinic for a routine checkup. He currently takes atorvastatin but is not achieving his target lipid profile. The physician prescribes colesevelam to help. Which of the following changes is expected as a result of this new therapy?
a) Decreased HDL
b) Decreased triglycerides
c) Hyperglycemia
d) Increased LDL
e) Increased triglycerides

A

[Y]Increased triglycerides

118
Q

(Question ID: 413) A 67-year-old woman with diabetes presents to clinic for an annual visit. She has been doing well over the past, and her hemoglobin A1c is 6.9%. Her fasting cholesterol showed a total cholesterol of 152 mg/dL, LDL of 68 mg/dL, and HDL of 31 mg/dL. Which medication is shown to have the greatest increase in HDL?
a) Cholestyramine
b) Ezetimibe
c) Gemfibrozil
d) Lovastatin
e) Niacin

A

[Y]Niacin

119
Q

(Question ID: 414) A 29-year-old man with a family history of heart disease comes to your office for a routine checkup. A lipid profile on a blood draw reveals high LDL and low HDL. One way to decrease the amount of LDL in the blood is to hinder intestinal absorption of cholesterol. Which of the following drugs blocks brush border enzymes to inhibit intestinal absorption of cholesterol?
a) Cholestyramine
b) Colesevelam
c) Colestipol
d) Ezetimibe
e) Rosuvastatin

A

[Y]Ezetimibe

120
Q

(Question ID: 415) A 47-year-old woman presents to the clinic for her annual visit. Her last fasting blood glucose on her last visit showed she has borderline diabetes. She tried to make lifestyle changes to improve her blood sugars. However, her fasting blood glucose is 215 mg/dL this year. She is started on metformin and encouraged to live a healthier lifestyle. What is a side effect associated with metformin?
a) Disulfiram-like reaction
b) Hypoglycemia
c) Lactic acidosis
d) Pancreatitis
e) Weight gain

A

[Y]Lactic acidosis

121
Q

(Question ID: 4244) A 36-year-old man from sub-Saharan Africa presents to the clinic with jaundice and right upper quadrant pain. On examination, the liver is palpably enlarged. Laboratory studies demonstrate an increase in liver enzymes. Computed tomography demonstrates a single large mass in the right lobe of the liver, and serum alfa-fetoprotein is markedly elevated. Which of the following is likely to have contributed to the patient’s condition?
a) Aflatoxin B1
b) Clonorchis sinensis
c) Hepatitis A
d) Polyvinyl chloride
e) Tetracycline

A

[Y]Aflatoxin B1